Limit points of $left{ frac{m}{2^{n}} right}$ where $m,n$ are natural numbers. [duplicate]












1












$begingroup$



This question already has an answer here:




  • The set of limit points of the sequence $1,frac12,frac14,frac34,frac18,frac38,frac58,frac78,frac1{16},frac3{16},ldots$

    3 answers




Let $E = left{frac{m}{2^{n}}right}$ where $m,n$ are integers, then $0$ is the only limit point of this set.



$($true$/$false$) ?$



I think the $0$ is the only limit point of this set. Since, as $n to infty$ terms of this sequence come nearer to $0$.



But the statement is false according to answer key.










share|cite|improve this question











$endgroup$



marked as duplicate by Cameron Buie, Lord Shark the Unknown, José Carlos Santos real-analysis
Users with the  real-analysis badge can single-handedly close real-analysis questions as duplicates and reopen them as needed.

StackExchange.ready(function() {
if (StackExchange.options.isMobile) return;

$('.dupe-hammer-message-hover:not(.hover-bound)').each(function() {
var $hover = $(this).addClass('hover-bound'),
$msg = $hover.siblings('.dupe-hammer-message');

$hover.hover(
function() {
$hover.showInfoMessage('', {
messageElement: $msg.clone().show(),
transient: false,
position: { my: 'bottom left', at: 'top center', offsetTop: -7 },
dismissable: false,
relativeToBody: true
});
},
function() {
StackExchange.helpers.removeMessages();
}
);
});
});
Dec 28 '18 at 8:54


This question has been asked before and already has an answer. If those answers do not fully address your question, please ask a new question.














  • 1




    $begingroup$
    The set ranges over all $n$ and $m$ so even if you take $n$ large you can compensate by taking $m$ even larger. See en.wikipedia.org/wiki/Dyadic_rational
    $endgroup$
    – Winther
    Dec 27 '18 at 12:41












  • $begingroup$
    Given a real number $x$ consider the sequence $a_k = frac{[x2^k]}{2^k}$ (i.e. $m = [x2^k]$ and $n = 2^k$ where $[cdot]$ is the closest integer function).
    $endgroup$
    – Winther
    Dec 27 '18 at 12:49












  • $begingroup$
    You may also want to consider my recent questions since they are similar to yours
    $endgroup$
    – roman
    Dec 27 '18 at 12:59












  • $begingroup$
    It is not too difficult to show that the set is dense in $[0,1]$, similar to this problem
    $endgroup$
    – rtybase
    Dec 27 '18 at 13:25
















1












$begingroup$



This question already has an answer here:




  • The set of limit points of the sequence $1,frac12,frac14,frac34,frac18,frac38,frac58,frac78,frac1{16},frac3{16},ldots$

    3 answers




Let $E = left{frac{m}{2^{n}}right}$ where $m,n$ are integers, then $0$ is the only limit point of this set.



$($true$/$false$) ?$



I think the $0$ is the only limit point of this set. Since, as $n to infty$ terms of this sequence come nearer to $0$.



But the statement is false according to answer key.










share|cite|improve this question











$endgroup$



marked as duplicate by Cameron Buie, Lord Shark the Unknown, José Carlos Santos real-analysis
Users with the  real-analysis badge can single-handedly close real-analysis questions as duplicates and reopen them as needed.

StackExchange.ready(function() {
if (StackExchange.options.isMobile) return;

$('.dupe-hammer-message-hover:not(.hover-bound)').each(function() {
var $hover = $(this).addClass('hover-bound'),
$msg = $hover.siblings('.dupe-hammer-message');

$hover.hover(
function() {
$hover.showInfoMessage('', {
messageElement: $msg.clone().show(),
transient: false,
position: { my: 'bottom left', at: 'top center', offsetTop: -7 },
dismissable: false,
relativeToBody: true
});
},
function() {
StackExchange.helpers.removeMessages();
}
);
});
});
Dec 28 '18 at 8:54


This question has been asked before and already has an answer. If those answers do not fully address your question, please ask a new question.














  • 1




    $begingroup$
    The set ranges over all $n$ and $m$ so even if you take $n$ large you can compensate by taking $m$ even larger. See en.wikipedia.org/wiki/Dyadic_rational
    $endgroup$
    – Winther
    Dec 27 '18 at 12:41












  • $begingroup$
    Given a real number $x$ consider the sequence $a_k = frac{[x2^k]}{2^k}$ (i.e. $m = [x2^k]$ and $n = 2^k$ where $[cdot]$ is the closest integer function).
    $endgroup$
    – Winther
    Dec 27 '18 at 12:49












  • $begingroup$
    You may also want to consider my recent questions since they are similar to yours
    $endgroup$
    – roman
    Dec 27 '18 at 12:59












  • $begingroup$
    It is not too difficult to show that the set is dense in $[0,1]$, similar to this problem
    $endgroup$
    – rtybase
    Dec 27 '18 at 13:25














1












1








1





$begingroup$



This question already has an answer here:




  • The set of limit points of the sequence $1,frac12,frac14,frac34,frac18,frac38,frac58,frac78,frac1{16},frac3{16},ldots$

    3 answers




Let $E = left{frac{m}{2^{n}}right}$ where $m,n$ are integers, then $0$ is the only limit point of this set.



$($true$/$false$) ?$



I think the $0$ is the only limit point of this set. Since, as $n to infty$ terms of this sequence come nearer to $0$.



But the statement is false according to answer key.










share|cite|improve this question











$endgroup$





This question already has an answer here:




  • The set of limit points of the sequence $1,frac12,frac14,frac34,frac18,frac38,frac58,frac78,frac1{16},frac3{16},ldots$

    3 answers




Let $E = left{frac{m}{2^{n}}right}$ where $m,n$ are integers, then $0$ is the only limit point of this set.



$($true$/$false$) ?$



I think the $0$ is the only limit point of this set. Since, as $n to infty$ terms of this sequence come nearer to $0$.



But the statement is false according to answer key.





This question already has an answer here:




  • The set of limit points of the sequence $1,frac12,frac14,frac34,frac18,frac38,frac58,frac78,frac1{16},frac3{16},ldots$

    3 answers








real-analysis






share|cite|improve this question















share|cite|improve this question













share|cite|improve this question




share|cite|improve this question








edited Dec 27 '18 at 13:07









Shaun

9,442113684




9,442113684










asked Dec 27 '18 at 12:38









MathsaddictMathsaddict

3669




3669




marked as duplicate by Cameron Buie, Lord Shark the Unknown, José Carlos Santos real-analysis
Users with the  real-analysis badge can single-handedly close real-analysis questions as duplicates and reopen them as needed.

StackExchange.ready(function() {
if (StackExchange.options.isMobile) return;

$('.dupe-hammer-message-hover:not(.hover-bound)').each(function() {
var $hover = $(this).addClass('hover-bound'),
$msg = $hover.siblings('.dupe-hammer-message');

$hover.hover(
function() {
$hover.showInfoMessage('', {
messageElement: $msg.clone().show(),
transient: false,
position: { my: 'bottom left', at: 'top center', offsetTop: -7 },
dismissable: false,
relativeToBody: true
});
},
function() {
StackExchange.helpers.removeMessages();
}
);
});
});
Dec 28 '18 at 8:54


This question has been asked before and already has an answer. If those answers do not fully address your question, please ask a new question.









marked as duplicate by Cameron Buie, Lord Shark the Unknown, José Carlos Santos real-analysis
Users with the  real-analysis badge can single-handedly close real-analysis questions as duplicates and reopen them as needed.

StackExchange.ready(function() {
if (StackExchange.options.isMobile) return;

$('.dupe-hammer-message-hover:not(.hover-bound)').each(function() {
var $hover = $(this).addClass('hover-bound'),
$msg = $hover.siblings('.dupe-hammer-message');

$hover.hover(
function() {
$hover.showInfoMessage('', {
messageElement: $msg.clone().show(),
transient: false,
position: { my: 'bottom left', at: 'top center', offsetTop: -7 },
dismissable: false,
relativeToBody: true
});
},
function() {
StackExchange.helpers.removeMessages();
}
);
});
});
Dec 28 '18 at 8:54


This question has been asked before and already has an answer. If those answers do not fully address your question, please ask a new question.










  • 1




    $begingroup$
    The set ranges over all $n$ and $m$ so even if you take $n$ large you can compensate by taking $m$ even larger. See en.wikipedia.org/wiki/Dyadic_rational
    $endgroup$
    – Winther
    Dec 27 '18 at 12:41












  • $begingroup$
    Given a real number $x$ consider the sequence $a_k = frac{[x2^k]}{2^k}$ (i.e. $m = [x2^k]$ and $n = 2^k$ where $[cdot]$ is the closest integer function).
    $endgroup$
    – Winther
    Dec 27 '18 at 12:49












  • $begingroup$
    You may also want to consider my recent questions since they are similar to yours
    $endgroup$
    – roman
    Dec 27 '18 at 12:59












  • $begingroup$
    It is not too difficult to show that the set is dense in $[0,1]$, similar to this problem
    $endgroup$
    – rtybase
    Dec 27 '18 at 13:25














  • 1




    $begingroup$
    The set ranges over all $n$ and $m$ so even if you take $n$ large you can compensate by taking $m$ even larger. See en.wikipedia.org/wiki/Dyadic_rational
    $endgroup$
    – Winther
    Dec 27 '18 at 12:41












  • $begingroup$
    Given a real number $x$ consider the sequence $a_k = frac{[x2^k]}{2^k}$ (i.e. $m = [x2^k]$ and $n = 2^k$ where $[cdot]$ is the closest integer function).
    $endgroup$
    – Winther
    Dec 27 '18 at 12:49












  • $begingroup$
    You may also want to consider my recent questions since they are similar to yours
    $endgroup$
    – roman
    Dec 27 '18 at 12:59












  • $begingroup$
    It is not too difficult to show that the set is dense in $[0,1]$, similar to this problem
    $endgroup$
    – rtybase
    Dec 27 '18 at 13:25








1




1




$begingroup$
The set ranges over all $n$ and $m$ so even if you take $n$ large you can compensate by taking $m$ even larger. See en.wikipedia.org/wiki/Dyadic_rational
$endgroup$
– Winther
Dec 27 '18 at 12:41






$begingroup$
The set ranges over all $n$ and $m$ so even if you take $n$ large you can compensate by taking $m$ even larger. See en.wikipedia.org/wiki/Dyadic_rational
$endgroup$
– Winther
Dec 27 '18 at 12:41














$begingroup$
Given a real number $x$ consider the sequence $a_k = frac{[x2^k]}{2^k}$ (i.e. $m = [x2^k]$ and $n = 2^k$ where $[cdot]$ is the closest integer function).
$endgroup$
– Winther
Dec 27 '18 at 12:49






$begingroup$
Given a real number $x$ consider the sequence $a_k = frac{[x2^k]}{2^k}$ (i.e. $m = [x2^k]$ and $n = 2^k$ where $[cdot]$ is the closest integer function).
$endgroup$
– Winther
Dec 27 '18 at 12:49














$begingroup$
You may also want to consider my recent questions since they are similar to yours
$endgroup$
– roman
Dec 27 '18 at 12:59






$begingroup$
You may also want to consider my recent questions since they are similar to yours
$endgroup$
– roman
Dec 27 '18 at 12:59














$begingroup$
It is not too difficult to show that the set is dense in $[0,1]$, similar to this problem
$endgroup$
– rtybase
Dec 27 '18 at 13:25




$begingroup$
It is not too difficult to show that the set is dense in $[0,1]$, similar to this problem
$endgroup$
– rtybase
Dec 27 '18 at 13:25










2 Answers
2






active

oldest

votes


















4












$begingroup$

Your set contains $$frac34,frac78,frac{15}{16},ldotsto 1$$so $1$ is a limit point.






share|cite|improve this answer









$endgroup$













  • $begingroup$
    Oh, yeah. If i take $m = 2^{n}-1$ then i will have this sequence converging to $1$. In general, this set has countably infinite limit points. Right$?$
    $endgroup$
    – Mathsaddict
    Dec 27 '18 at 12:49












  • $begingroup$
    @Mathsaddict this set is dense in $mathbb R$ (i,e every point is a limit point)
    $endgroup$
    – Anvit
    Dec 27 '18 at 12:53












  • $begingroup$
    @Anvit then this set has uncountable limit points!
    $endgroup$
    – Mathsaddict
    Dec 27 '18 at 12:57










  • $begingroup$
    @Mathsaddict, Yep, thats correct. Check the link in second comment on your post.
    $endgroup$
    – Anvit
    Dec 27 '18 at 12:58





















1












$begingroup$

It is not too difficult to show that the set is dense, similar to this problem.



We can generalise the problem ...






Propositions 1. $forall r in [0,infty)$ and $forall varepsilon>0, exists m,n in mathbb{N}: left|r-frac{m}{q^n}right|<varepsilon$, for a fixed $qin mathbb{N}setminus {0,1}$.




From
$$color{red}{0leq m}=left lfloor rcdot q^n right rfloor leq rcdot q^n < left lfloor rcdot q^n right rfloor +1=m+1 Rightarrow
frac{m}{q^n} leq r < frac{m}{q^n}+frac{1}{q^n}$$

we have
$$left|r-frac{m}{q^n}right|<frac{1}{q^n}$$
By "adjusting" $n>0$ we can find one such that $frac{1}{q^n} < varepsilon$. Also, $m,n in mathbb{N}$.



NOTE: This means that $M_q=left{frac{m}{q^n} mid m,ninmathbb{N}right}$, $qin mathbb{N}setminus {0,1}$, is dense in $[0,infty)$ or any point in $[0,infty)$ is a limit point for $M_q$.






Proposition 2 $left{left{frac{m}{q^n}right} mid m,ninmathbb{N}right}$, for a fixed $qin mathbb{N}setminus {0,1}$, is dense in $[0,1]$.



(the second ${}$ is the fractional part).




For $forall r in [0,1)$ and $forall varepsilon >0$ s.t. $0leq r < r+ varepsilon < 1$, we always have (from proposition 1) $m,n inmathbb{N}$ s.t.
$$color{red}{0}leq r leq color{red}{frac{m}{q^n}} < r+ varepsilon < color{red}{1}$$
(because if $M_q$ is dense in $[0,infty)$, then it is also dense in $[0,1) subset [0,infty)$. But this means (see the parts highlighted in red) that $frac{m}{q^n} = left{frac{m}{q^n}right}$ (${}$ is the fractional part). The only point left to check is ${1}$. For this purpose, we will be looking at
$$r_k=1-frac{1}{k} < 1-frac{1}{k} + varepsilon < 1$$
And again, (from proposition 1) we have $m_k,n_k inmathbb{N}$ s.t.
$$r_k=1-frac{1}{k} leq frac{m_k}{q_k^n} < 1-frac{1}{k} + varepsilon < 1$$
or
$$left|1-frac{m_k}{q_k^n}right|leq left|1-r_kright|=frac{1}{k}$$
which, again, can be made as small as we want and we covered ${1}$ too and thus the entire $[0,1] = [0,1) cup {1}$





Summarising all these and making $q=2$ the answer is obviously false.






share|cite|improve this answer











$endgroup$




















    2 Answers
    2






    active

    oldest

    votes








    2 Answers
    2






    active

    oldest

    votes









    active

    oldest

    votes






    active

    oldest

    votes









    4












    $begingroup$

    Your set contains $$frac34,frac78,frac{15}{16},ldotsto 1$$so $1$ is a limit point.






    share|cite|improve this answer









    $endgroup$













    • $begingroup$
      Oh, yeah. If i take $m = 2^{n}-1$ then i will have this sequence converging to $1$. In general, this set has countably infinite limit points. Right$?$
      $endgroup$
      – Mathsaddict
      Dec 27 '18 at 12:49












    • $begingroup$
      @Mathsaddict this set is dense in $mathbb R$ (i,e every point is a limit point)
      $endgroup$
      – Anvit
      Dec 27 '18 at 12:53












    • $begingroup$
      @Anvit then this set has uncountable limit points!
      $endgroup$
      – Mathsaddict
      Dec 27 '18 at 12:57










    • $begingroup$
      @Mathsaddict, Yep, thats correct. Check the link in second comment on your post.
      $endgroup$
      – Anvit
      Dec 27 '18 at 12:58


















    4












    $begingroup$

    Your set contains $$frac34,frac78,frac{15}{16},ldotsto 1$$so $1$ is a limit point.






    share|cite|improve this answer









    $endgroup$













    • $begingroup$
      Oh, yeah. If i take $m = 2^{n}-1$ then i will have this sequence converging to $1$. In general, this set has countably infinite limit points. Right$?$
      $endgroup$
      – Mathsaddict
      Dec 27 '18 at 12:49












    • $begingroup$
      @Mathsaddict this set is dense in $mathbb R$ (i,e every point is a limit point)
      $endgroup$
      – Anvit
      Dec 27 '18 at 12:53












    • $begingroup$
      @Anvit then this set has uncountable limit points!
      $endgroup$
      – Mathsaddict
      Dec 27 '18 at 12:57










    • $begingroup$
      @Mathsaddict, Yep, thats correct. Check the link in second comment on your post.
      $endgroup$
      – Anvit
      Dec 27 '18 at 12:58
















    4












    4








    4





    $begingroup$

    Your set contains $$frac34,frac78,frac{15}{16},ldotsto 1$$so $1$ is a limit point.






    share|cite|improve this answer









    $endgroup$



    Your set contains $$frac34,frac78,frac{15}{16},ldotsto 1$$so $1$ is a limit point.







    share|cite|improve this answer












    share|cite|improve this answer



    share|cite|improve this answer










    answered Dec 27 '18 at 12:43









    ArthurArthur

    118k7117200




    118k7117200












    • $begingroup$
      Oh, yeah. If i take $m = 2^{n}-1$ then i will have this sequence converging to $1$. In general, this set has countably infinite limit points. Right$?$
      $endgroup$
      – Mathsaddict
      Dec 27 '18 at 12:49












    • $begingroup$
      @Mathsaddict this set is dense in $mathbb R$ (i,e every point is a limit point)
      $endgroup$
      – Anvit
      Dec 27 '18 at 12:53












    • $begingroup$
      @Anvit then this set has uncountable limit points!
      $endgroup$
      – Mathsaddict
      Dec 27 '18 at 12:57










    • $begingroup$
      @Mathsaddict, Yep, thats correct. Check the link in second comment on your post.
      $endgroup$
      – Anvit
      Dec 27 '18 at 12:58




















    • $begingroup$
      Oh, yeah. If i take $m = 2^{n}-1$ then i will have this sequence converging to $1$. In general, this set has countably infinite limit points. Right$?$
      $endgroup$
      – Mathsaddict
      Dec 27 '18 at 12:49












    • $begingroup$
      @Mathsaddict this set is dense in $mathbb R$ (i,e every point is a limit point)
      $endgroup$
      – Anvit
      Dec 27 '18 at 12:53












    • $begingroup$
      @Anvit then this set has uncountable limit points!
      $endgroup$
      – Mathsaddict
      Dec 27 '18 at 12:57










    • $begingroup$
      @Mathsaddict, Yep, thats correct. Check the link in second comment on your post.
      $endgroup$
      – Anvit
      Dec 27 '18 at 12:58


















    $begingroup$
    Oh, yeah. If i take $m = 2^{n}-1$ then i will have this sequence converging to $1$. In general, this set has countably infinite limit points. Right$?$
    $endgroup$
    – Mathsaddict
    Dec 27 '18 at 12:49






    $begingroup$
    Oh, yeah. If i take $m = 2^{n}-1$ then i will have this sequence converging to $1$. In general, this set has countably infinite limit points. Right$?$
    $endgroup$
    – Mathsaddict
    Dec 27 '18 at 12:49














    $begingroup$
    @Mathsaddict this set is dense in $mathbb R$ (i,e every point is a limit point)
    $endgroup$
    – Anvit
    Dec 27 '18 at 12:53






    $begingroup$
    @Mathsaddict this set is dense in $mathbb R$ (i,e every point is a limit point)
    $endgroup$
    – Anvit
    Dec 27 '18 at 12:53














    $begingroup$
    @Anvit then this set has uncountable limit points!
    $endgroup$
    – Mathsaddict
    Dec 27 '18 at 12:57




    $begingroup$
    @Anvit then this set has uncountable limit points!
    $endgroup$
    – Mathsaddict
    Dec 27 '18 at 12:57












    $begingroup$
    @Mathsaddict, Yep, thats correct. Check the link in second comment on your post.
    $endgroup$
    – Anvit
    Dec 27 '18 at 12:58






    $begingroup$
    @Mathsaddict, Yep, thats correct. Check the link in second comment on your post.
    $endgroup$
    – Anvit
    Dec 27 '18 at 12:58













    1












    $begingroup$

    It is not too difficult to show that the set is dense, similar to this problem.



    We can generalise the problem ...






    Propositions 1. $forall r in [0,infty)$ and $forall varepsilon>0, exists m,n in mathbb{N}: left|r-frac{m}{q^n}right|<varepsilon$, for a fixed $qin mathbb{N}setminus {0,1}$.




    From
    $$color{red}{0leq m}=left lfloor rcdot q^n right rfloor leq rcdot q^n < left lfloor rcdot q^n right rfloor +1=m+1 Rightarrow
    frac{m}{q^n} leq r < frac{m}{q^n}+frac{1}{q^n}$$

    we have
    $$left|r-frac{m}{q^n}right|<frac{1}{q^n}$$
    By "adjusting" $n>0$ we can find one such that $frac{1}{q^n} < varepsilon$. Also, $m,n in mathbb{N}$.



    NOTE: This means that $M_q=left{frac{m}{q^n} mid m,ninmathbb{N}right}$, $qin mathbb{N}setminus {0,1}$, is dense in $[0,infty)$ or any point in $[0,infty)$ is a limit point for $M_q$.






    Proposition 2 $left{left{frac{m}{q^n}right} mid m,ninmathbb{N}right}$, for a fixed $qin mathbb{N}setminus {0,1}$, is dense in $[0,1]$.



    (the second ${}$ is the fractional part).




    For $forall r in [0,1)$ and $forall varepsilon >0$ s.t. $0leq r < r+ varepsilon < 1$, we always have (from proposition 1) $m,n inmathbb{N}$ s.t.
    $$color{red}{0}leq r leq color{red}{frac{m}{q^n}} < r+ varepsilon < color{red}{1}$$
    (because if $M_q$ is dense in $[0,infty)$, then it is also dense in $[0,1) subset [0,infty)$. But this means (see the parts highlighted in red) that $frac{m}{q^n} = left{frac{m}{q^n}right}$ (${}$ is the fractional part). The only point left to check is ${1}$. For this purpose, we will be looking at
    $$r_k=1-frac{1}{k} < 1-frac{1}{k} + varepsilon < 1$$
    And again, (from proposition 1) we have $m_k,n_k inmathbb{N}$ s.t.
    $$r_k=1-frac{1}{k} leq frac{m_k}{q_k^n} < 1-frac{1}{k} + varepsilon < 1$$
    or
    $$left|1-frac{m_k}{q_k^n}right|leq left|1-r_kright|=frac{1}{k}$$
    which, again, can be made as small as we want and we covered ${1}$ too and thus the entire $[0,1] = [0,1) cup {1}$





    Summarising all these and making $q=2$ the answer is obviously false.






    share|cite|improve this answer











    $endgroup$


















      1












      $begingroup$

      It is not too difficult to show that the set is dense, similar to this problem.



      We can generalise the problem ...






      Propositions 1. $forall r in [0,infty)$ and $forall varepsilon>0, exists m,n in mathbb{N}: left|r-frac{m}{q^n}right|<varepsilon$, for a fixed $qin mathbb{N}setminus {0,1}$.




      From
      $$color{red}{0leq m}=left lfloor rcdot q^n right rfloor leq rcdot q^n < left lfloor rcdot q^n right rfloor +1=m+1 Rightarrow
      frac{m}{q^n} leq r < frac{m}{q^n}+frac{1}{q^n}$$

      we have
      $$left|r-frac{m}{q^n}right|<frac{1}{q^n}$$
      By "adjusting" $n>0$ we can find one such that $frac{1}{q^n} < varepsilon$. Also, $m,n in mathbb{N}$.



      NOTE: This means that $M_q=left{frac{m}{q^n} mid m,ninmathbb{N}right}$, $qin mathbb{N}setminus {0,1}$, is dense in $[0,infty)$ or any point in $[0,infty)$ is a limit point for $M_q$.






      Proposition 2 $left{left{frac{m}{q^n}right} mid m,ninmathbb{N}right}$, for a fixed $qin mathbb{N}setminus {0,1}$, is dense in $[0,1]$.



      (the second ${}$ is the fractional part).




      For $forall r in [0,1)$ and $forall varepsilon >0$ s.t. $0leq r < r+ varepsilon < 1$, we always have (from proposition 1) $m,n inmathbb{N}$ s.t.
      $$color{red}{0}leq r leq color{red}{frac{m}{q^n}} < r+ varepsilon < color{red}{1}$$
      (because if $M_q$ is dense in $[0,infty)$, then it is also dense in $[0,1) subset [0,infty)$. But this means (see the parts highlighted in red) that $frac{m}{q^n} = left{frac{m}{q^n}right}$ (${}$ is the fractional part). The only point left to check is ${1}$. For this purpose, we will be looking at
      $$r_k=1-frac{1}{k} < 1-frac{1}{k} + varepsilon < 1$$
      And again, (from proposition 1) we have $m_k,n_k inmathbb{N}$ s.t.
      $$r_k=1-frac{1}{k} leq frac{m_k}{q_k^n} < 1-frac{1}{k} + varepsilon < 1$$
      or
      $$left|1-frac{m_k}{q_k^n}right|leq left|1-r_kright|=frac{1}{k}$$
      which, again, can be made as small as we want and we covered ${1}$ too and thus the entire $[0,1] = [0,1) cup {1}$





      Summarising all these and making $q=2$ the answer is obviously false.






      share|cite|improve this answer











      $endgroup$
















        1












        1








        1





        $begingroup$

        It is not too difficult to show that the set is dense, similar to this problem.



        We can generalise the problem ...






        Propositions 1. $forall r in [0,infty)$ and $forall varepsilon>0, exists m,n in mathbb{N}: left|r-frac{m}{q^n}right|<varepsilon$, for a fixed $qin mathbb{N}setminus {0,1}$.




        From
        $$color{red}{0leq m}=left lfloor rcdot q^n right rfloor leq rcdot q^n < left lfloor rcdot q^n right rfloor +1=m+1 Rightarrow
        frac{m}{q^n} leq r < frac{m}{q^n}+frac{1}{q^n}$$

        we have
        $$left|r-frac{m}{q^n}right|<frac{1}{q^n}$$
        By "adjusting" $n>0$ we can find one such that $frac{1}{q^n} < varepsilon$. Also, $m,n in mathbb{N}$.



        NOTE: This means that $M_q=left{frac{m}{q^n} mid m,ninmathbb{N}right}$, $qin mathbb{N}setminus {0,1}$, is dense in $[0,infty)$ or any point in $[0,infty)$ is a limit point for $M_q$.






        Proposition 2 $left{left{frac{m}{q^n}right} mid m,ninmathbb{N}right}$, for a fixed $qin mathbb{N}setminus {0,1}$, is dense in $[0,1]$.



        (the second ${}$ is the fractional part).




        For $forall r in [0,1)$ and $forall varepsilon >0$ s.t. $0leq r < r+ varepsilon < 1$, we always have (from proposition 1) $m,n inmathbb{N}$ s.t.
        $$color{red}{0}leq r leq color{red}{frac{m}{q^n}} < r+ varepsilon < color{red}{1}$$
        (because if $M_q$ is dense in $[0,infty)$, then it is also dense in $[0,1) subset [0,infty)$. But this means (see the parts highlighted in red) that $frac{m}{q^n} = left{frac{m}{q^n}right}$ (${}$ is the fractional part). The only point left to check is ${1}$. For this purpose, we will be looking at
        $$r_k=1-frac{1}{k} < 1-frac{1}{k} + varepsilon < 1$$
        And again, (from proposition 1) we have $m_k,n_k inmathbb{N}$ s.t.
        $$r_k=1-frac{1}{k} leq frac{m_k}{q_k^n} < 1-frac{1}{k} + varepsilon < 1$$
        or
        $$left|1-frac{m_k}{q_k^n}right|leq left|1-r_kright|=frac{1}{k}$$
        which, again, can be made as small as we want and we covered ${1}$ too and thus the entire $[0,1] = [0,1) cup {1}$





        Summarising all these and making $q=2$ the answer is obviously false.






        share|cite|improve this answer











        $endgroup$



        It is not too difficult to show that the set is dense, similar to this problem.



        We can generalise the problem ...






        Propositions 1. $forall r in [0,infty)$ and $forall varepsilon>0, exists m,n in mathbb{N}: left|r-frac{m}{q^n}right|<varepsilon$, for a fixed $qin mathbb{N}setminus {0,1}$.




        From
        $$color{red}{0leq m}=left lfloor rcdot q^n right rfloor leq rcdot q^n < left lfloor rcdot q^n right rfloor +1=m+1 Rightarrow
        frac{m}{q^n} leq r < frac{m}{q^n}+frac{1}{q^n}$$

        we have
        $$left|r-frac{m}{q^n}right|<frac{1}{q^n}$$
        By "adjusting" $n>0$ we can find one such that $frac{1}{q^n} < varepsilon$. Also, $m,n in mathbb{N}$.



        NOTE: This means that $M_q=left{frac{m}{q^n} mid m,ninmathbb{N}right}$, $qin mathbb{N}setminus {0,1}$, is dense in $[0,infty)$ or any point in $[0,infty)$ is a limit point for $M_q$.






        Proposition 2 $left{left{frac{m}{q^n}right} mid m,ninmathbb{N}right}$, for a fixed $qin mathbb{N}setminus {0,1}$, is dense in $[0,1]$.



        (the second ${}$ is the fractional part).




        For $forall r in [0,1)$ and $forall varepsilon >0$ s.t. $0leq r < r+ varepsilon < 1$, we always have (from proposition 1) $m,n inmathbb{N}$ s.t.
        $$color{red}{0}leq r leq color{red}{frac{m}{q^n}} < r+ varepsilon < color{red}{1}$$
        (because if $M_q$ is dense in $[0,infty)$, then it is also dense in $[0,1) subset [0,infty)$. But this means (see the parts highlighted in red) that $frac{m}{q^n} = left{frac{m}{q^n}right}$ (${}$ is the fractional part). The only point left to check is ${1}$. For this purpose, we will be looking at
        $$r_k=1-frac{1}{k} < 1-frac{1}{k} + varepsilon < 1$$
        And again, (from proposition 1) we have $m_k,n_k inmathbb{N}$ s.t.
        $$r_k=1-frac{1}{k} leq frac{m_k}{q_k^n} < 1-frac{1}{k} + varepsilon < 1$$
        or
        $$left|1-frac{m_k}{q_k^n}right|leq left|1-r_kright|=frac{1}{k}$$
        which, again, can be made as small as we want and we covered ${1}$ too and thus the entire $[0,1] = [0,1) cup {1}$





        Summarising all these and making $q=2$ the answer is obviously false.







        share|cite|improve this answer














        share|cite|improve this answer



        share|cite|improve this answer








        edited Dec 27 '18 at 14:43

























        answered Dec 27 '18 at 14:38









        rtybasertybase

        11.4k31533




        11.4k31533















            Popular posts from this blog

            To store a contact into the json file from server.js file using a class in NodeJS

            Redirect URL with Chrome Remote Debugging Android Devices

            Dieringhausen